LSAT and Law School Admissions Forum

Get expert LSAT preparation and law school admissions advice from PowerScore Test Preparation.

Explanations for the questions in the lesson portion of the Accelerated Course materials.
 Jon Denning
PowerScore Staff
  • PowerScore Staff
  • Posts: 904
  • Joined: Apr 11, 2011
|
#38834
Taken from: lsat/viewtopic.php?f=606&t=11052

WeakenX. The correct answer choice is (C)

The argument presented in this stimulus sounds reasonable, although the word “established” makes this a fairly strong assertion. The referenced studies may suggest that people would want to be informed, but this is different from establishing (or proving) the claim irrefutably. Since this is an Except question, four of the answer choices will weaken the author’s argument, and one will not (note that the correct answer may not strengthen—we only know that it won’t weaken the argument—it could be irrelevant).

Answer choice (A): If another study provides such a contrary opinion, then this certainly weakens the author’s claim. Since this answer choice does weaken the argument in the stimulus, so it is an incorrect response to this Except question.

Answer choice (B): This answer choice gives us reason to question the results of the referenced survey, so this is another incorrect response to this Weaken Except question.

Answer choice (C): This is the correct answer choice. Such a study would not require a medical background to assess the results. Since this choice does not weaken the conclusion in the stimulus, this is the right answer.

Answer choice (D): If some of the questions suggested that wanting to be informed is a reasonable response, such questions are biased and would decrease the credibility of the referenced studies.

Answer choice (E): If every single survey respondent was a student of introductory psychology, there is a clear possibility of bias in the population (this group may be particularly curious, for example). With such a biased population, there is reason to question the results of the studied cited. Since this answer also weakens the author’s argument, this is incorrect response to this Weaken Except question.

Get the most out of your LSAT Prep Plus subscription.

Analyze and track your performance with our Testing and Analytics Package.